Prep U exam 2

¡Supera tus tareas y exámenes ahora con Quizwiz!

The nurse is conducting a health history with a female client who reports upper back and jaw pain. In order to assess the client's risk for a cardiac event, which question should the nurse ask first? "Do you have any pain or discomfort in your chest?" "Is the pain worse on exertion?" "Do you have cramping pain?' "Is the pain worse when you are lying down?"

"Do you have any pain or discomfort in your chest?"

A client presents with otalgia and yellow-green discharge from the external ear canal. Which question should the nurse ask to determine the cause of this problem? "Have you had any recent trauma to the inside of your ear?" "Do you hear ringing in your ears?" "Are their times when you feel dizzy?" "Have you ever taken medication that is ototoxic?"

"Have you had any recent trauma to the inside of your ear?"

A nursing instructor is teaching students about communication in different cultures. When discussing the meaning of hand gestures and body language in different cultures, the nurse realizes that further instruction is necessary when a student makes which statement? "There are many different elements of body language and hand gestures." "If any hand gesture is used, always clarify if there seems to be a strange reaction on the other's part." "I can make a circle with my thumb and forefinger and people of all cultures know it means OK." "A major hand gesture is one for indicating height."

"I can make a circle with my thumb and forefinger and people of all cultures know it means OK."

Which statement by the nurse demonstrates an understanding of the importance that a client's culture plays in the client's health and wellness? "I need to understand the client's cultural background to best interpret the client's needs." "A person's race is an effective characteristic of his or her cultural background." "I understand a client's culture if I am familiar with the client's ethnic values." "A disability is not a factor in determining a client's cultural needs."

"I need to understand the client's cultural background to best interpret the client's needs."

The client states his pain is "intolerable" and requests pain medication. The nurse observes the client to be talking and laughing with visitors. How should the nurse best respond? "I will be back with your pain medication." "If we can get some visitors out of your room, you might rest better." "I cannot imagine you are having pain the way you are laughing in here." "You do not look like you are in pain."

"I will be back with your pain medication."

The client asks the nurse why the nurse put the tuning fork on the bone behind the ear. Which is the best response by the nurse? "It identifies a problem with the normal pathways for sound to travel to your inner ear." "It can identify if you have an inner ear problem causing disequilibrium." "It determines hearing loss caused by degeneration of nerves in your inner ear." "It can determine if you have a problem with repeated ear infections."

"It identifies a problem with the normal pathways for sound to travel to your inner ear."

An adult client tells the nurse that his 80-year-old father is almost completely deaf. After an explanation to the client about risk factors for hearing loss, the nurse determines that the client needs further instruction when the client says "There is a genetic predisposition to hearing loss." "Certain cultural groups have a higher rate of hearing loss." "It is difficult to prevent hearing loss or worsening of hearing." "Chronic otitis media has been associated with hearing loss."

"It is difficult to prevent hearing loss or worsening of hearing."

A 52-year-old client with myopia calls the ophthalmology clinic very upset. She tells the nurse, "I keep seeing semi-clear spots floating across my vision. What is wrong with me?" What would be the most appropriate response by the nurse? "It is not an uncommon finding in people older than 40 years for this to happen. They are called 'floaters'." "Please come into the clinic right away so we can see what is wrong." "Because it is almost 5 o'clock, please go to the emergency department right away. This sounds very serious." "I have an opening tomorrow at 2 in the afternoon. Can you come in then?"

"It is not an uncommon finding in people older than 40 years for this to happen. They are called 'floaters'."

Which statement by the nurse demonstrates a cultural bias? "I don't understand how a woman would be sexually interested in another woman." "The client is Muslim, so I'll need to arrange care around his need to pray five times a day." "American healthcare is really so much better than that provided by any other country." "The client is old and wouldn't be able to understand the discharge instructions."

"American healthcare is really so much better than that provided by any other country."

A client is diagnosed with a scotoma. What question is appropriate for the nurse to ask to obtain more data about this condition? "Do you see floaters in front of your eyes?" "Are the blind spots constant or intermittent?" "How often do you have redness or tearing?" "Is night blindness a problem for you?"

"Are the blind spots constant or intermittent?"

A nurse is interviewing a client as part of a routine examination of his ears and hearing. The nurse notes that this client has high blood pressure. Which of the following questions regarding his hearing should the nurse ask that is associated with his high blood pressure? "Do you have any ear pain?" "Do you experience any ringing, roaring, or crackling in your ears?" "Do you have any ear drainage?" "Are you ever concerned that you may be losing your ability to hear well?"

"Do you experience any ringing, roaring, or crackling in your ears?"

What open-ended question might be helpful when assessing abuse with a client demonstrating discomfort discussing the issue? "What can I tell you about abuse?" "I know you have been abused. Please tell me about it." "Abuse is more prevalent than most people think." "What would you like me to know?"

"What would you like me to know?"

The results of a client's Rinne test are as follows: bone conduction > air conduction. How should the nurse explain these findings to the client? "You have a high frequency hearing loss." "You have a conductive hearing loss." "You have nerve damage in your ears." "You have a unilateral hearing loss."

"You have a conductive hearing loss."

A nurse examines a client with complaints of a sore throat and finds that the tonsils are just visible. Using a grading scale of 1+ to 4+, how should the nurse appropriately document the tonsils? 1+ 2+ 3+ 4+

1+

Which reading of the ankle-brachial pressure index (ABPI) should the nurse recognize as indicative of a normal healthy person? 0.15 0.25 0.75 1.00

1.00

The nurse assesses the client's pulses to be normal. How would the nurse document this information? 0 1+ 2+ 4+

2+

The nurse begins auscultating a client's heart sounds at the 2nd intercostal space right sternal border. Which location should the nurse assess next? 4th left intercostal space 3rd intercostal space left sternal border 2nd intercostal space left sternal border 5th left intercostal space midclavicular line

2nd intercostal space left sternal border

A nurse assesses a client who was physically assaulted by his brother. The nurse learns that the client has been punched and kicked. After marking the area of injury on the body map, what score should the nurse document for this abuse? 1 3 4 5

3

Where is Erb's point located? 4th left rib space 3rd right rib space 4th right rib space 3rd left rib space

3rd left rib space

An increased risk of falls is dangerous for any client. What client would be at an increased risk of falls? A client with a hearing loss of 45 dB. A client with acute otitis media. A client with vertigo. A client with damage to the VIIIth cranial nerve.

A client with vertigo.

From a critical cultural perspective, culture refers to which of the following? Values, beliefs, and practices of specific groups Genetically inherited behavioral traits A web of connections among ethnically-related persons A dynamic process enacted between people and their families

A dynamic process enacted between people and their families

A nurse has a regular client who is an immigrant from China and who follows the traditional medical system of that culture. To improve the ability to understand and work with this client, the nurse researches this client's cultural view of health. Which of the following would the nurse most likely discover? A focus on maintaining balance between yin and yang Respect for nature and use of masks and sand paintings Prayers to God and saints for spiritual reparations for sins Use of herbs, roots, talismans, and amulets

A focus on maintaining balance between yin and yang

The nurse notes a tophus of the ear of an older adult. Which assessment data is consistent with a tophus? A hard nodule composed of uric acid crystals A sac with a membranous lining filled with fluid Scarring of the tympanic membrane Redness and bulging of the eardrum

A hard nodule composed of uric acid crystals

A nurse finds crepitus when palpating over a client's maxillary sinuses. Which of the following should the nurse most suspect in this client? Normal, air-filled sinuses A large amount of exudate in the sinuses Obstruction of the nostril by a foreign object A perforated septum

A large amount of exudate in the sinuses

Which of the following wounds is most likely attributable to neuropathy? A painful wound in the client's shin, which is surrounded by apparently healthy skin A moderately painful wound on the lateral aspect of the client's ankle A painless wound on the sole of the client's foot, which is surrounded by calloused skin A wound on a client's highly edematous ankle that is surrounded by pigmented skin

A painless wound on the sole of the client's foot, which is surrounded by calloused skin

A nurse is counseling a woman who has been abused by her husband and who is at risk for serious injury and even death. The woman has two school-aged children at home. Which of the following should the nurse tell the woman to make sure her children know? Select all that apply. A safe place to go when abuse of the client occurs Who is safe to tell when they are unsafe That it is their job to keep their mother safe How they should intervene during abuse to make it stop How and when to call 911

A safe place to go when abuse of the client occurs Who is safe to tell when they are unsafe How and when to call 911

A client has Darwin tubercle. What is this? A type of skin cancer found on the ear A growth in the ear canal A growth in the bony labyrinth A small painless nodule on the helix

A small painless nodule on the helix

A student states that a client has palpable rushing vibration in the area of the pulmonic valve. What should the instructor explain that the student is feeling? A thrill A thrust A heave A normal finding

A thrill

A nurse palpates the presence of an enlarged inguinal lymph node. Which area of the client's body should the nurse thoroughly examine to assess for the source of this finding? Cervical lymph nodes for tenderness and swelling Abdomen, noting any organ enlargement or tenderness Head and neck for recent ear infection or sore throat Lower arm and hand for erythema and swelling

Abdomen, noting any organ enlargement or tenderness

Where is the temporal artery palpated? Above the cheek bone near the scalp line Just left of midline at the base of the neck Between the mandibular joint and the base of the ear Just left or right of the spine at the base of the skull

Above the cheek bone near the scalp line

The nurse assesses the frontal sinus where? Above the eyes Below the eyes Above jaw Below jaw

Above the eyes

When auscultating the heart sounds of a client, a nurse notes that the S2 is louder than the S1. How should the nurse describe S2? Accentuated Diminished Normal split Wide split

Accentuated

A client with cancer is talking to the primary nurse about the diagnosis and states, "I feel like it is God's will that I have this cancer. If I get out of the hospital, I want to take nature walks like I used to before I got so sick. I feel so close to God when I do this." The nurse understands that this client is demonstrating which of the following? Traumatization over this diagnosis Acceptance of the diagnosis Viewing the cancer as punishment for past lifestyle Signs of depression

Acceptance of the diagnosis

A client presents to the health care clinic with reports of a 3-day history of fever, sore throat, and trouble swallowing. The nurse notes the client to be febrile with temperature 101.5° F, tonsils are 2+ and red, transillumination of the sinuses is normal. Which nursing diagnosis should the nurse confirm based on this data? Impaired Neurological or Neuromuscular Function Acute Pain Ineffective Health Maintenance Self-Care Deficit

Acute Pain

Which factor, if present in a client's lifestyle and health practices assessment, would alert the nurse to the need for performing a more thorough head and neck assessment? Select all that apply. Alcohol abuse Recreational drug use Smokeless tobacco use Multiple sex partners

Alcohol abuse Recreational drug use Smokeless tobacco use Multiple sex partners

Which of the following is true of psychological abuse? Psychological abuse is easy to define and diagnose. An abuser may use psychological behaviors such as belittling, exploiting, denigrating, or remaining emotionally unresponsive. The majority of children experiencing psychological abuse do not use effective coping mechanisms. Psychological abuse of children does not affect long-term development.

An abuser may use psychological behaviors such as belittling, exploiting, denigrating, or remaining emotionally unresponsive.

The Kiesselbach plexus is the most common site for what? Anterior nosebleeds Posterior nosebleeds Sinusitis Infections

Anterior nosebleeds

When collecting a client's medical history, the nurse should ask if the client is taking which medications that most likely contribute to complaints of recurrent epistaxis? (Select all that apply.) Anticoagulants Herbal supplements Antihistamines Central nervous system agents Hormones

Anticoagulants Herbal supplements Antihistamines

A 82 year old female presents with neck pain, decreased strength and sensation of the upper extremities. The nurse identifies that this could be related to what? Arthritic changes of the cervical spine Bacterial thyroiditis Cranial damage Muscle tension

Arthritic changes of the cervical spine

A nurse is preparing to examine a client from Southeast Asia who has been experiencing chronic headaches. Which of the following should the nurse do in light of this client's cultural background? Avoid asking the client to remove her clothes for the examination Have a nurse who is the same sex as the client perform the examination Ask permission before palpating the head and neck Palpate the client's feet before palpating the head

Ask permission before palpating the head and neck

A mother of a small child calls the clinic and asks to schedule an appointment for ear tube removal. The call is transferred to the nurse. What is the nurse's best action? Schedule first available office appointment. Schedule appointment at hospital for tubes to be removed surgically. Ask healthcare provider about prescribing antibiotics before removal. Ask the mother how long the tubes have been in place.

Ask the mother how long the tubes have been in place.

Which approach would be most appropriate when counseling a woman who is a suspected victim of violence? Offer her a pamphlet about the local battered women's shelter. Call her home to ask her some questions about her marriage. Wait until she comes in a few more times to make a better assessment. Ask, "Have you ever been physically hurt by your partner?"

Ask, have you eve been physically hurt by your partner

Which technique by the nurse demonstrates proper use of the ophthalmoscope? Uses right eye to examine the client's left eye Moves the scope around so the entire optic disk may be seen Approaches the client directly in front of the pupil Asks the client to fix the gaze upon an object and look straight ahead

Asks the client to fix the gaze upon an object and look straight ahead

How should a nurse assess a client for pulse rate deficit? Assess for a difference between the apical and radial pulse Check for pulse inequality between right and left carotid arteries Auscultate for split S1 at the base and apex Observe for a decrease in jugular venous pressure

Assess for a difference between the apical and radial pulse

A nurse cares for a client who is postoperative cholecystectomy. Which action by the nurse is appropriate to help prevent the occurrence of venous stasis? Raise the foot of the bed for an hour and then lower it. Massage lower extremities vigorously every 6 hours. Assist in active range-of-motion exercise of the upper body. Assist the client to walk as soon and as often as possible.

Assist the client to walk as soon and as often as possible.

A client arrives complaining of nasal congestion, drainage of a thick, yellow discharge from the nose, difficulty breathing through the nose, headache, and pressure in the forehead. The nurse suspects sinusitis. Which of the following risk factors should the nurse assess for in this client? Chewing betel nuts Exposure to the sun Asthma Heavy alcohol use

Asthma

As a part of the ear examination for hearing loss, a nurse conducts a Weber test on a client. To accurately perform this test, the nurse should place the base of the tuning fork in which of the following locations? At the center of the client's forehead On the client's mastoid process In front of the external auditory canal Behind the external auditory canal

At the center of the client's forehead

When providing client education on hearing, the nurse should remind clients to utilize ear plugs when they are what? (Select all that apply.) At train stations Cleaning their homes Using lawnmowers Working with children At concerts

At train stations Using lawnmowers At concerts

When inspecting the nose with an otoscope, the nurse: Holds the handle vertically Uses the smallest speculum available Avoids contact with the nasal septum Directs the nasal speculum superiorly and medially

Avoids contact with the nasal septum

The nurse helps with a community health fair and suspects that one young girl may be a victim of human trafficking. What did the nurse observe to come to this conclusion? Select all that apply. Avoids eye contact Provides a street address Wears clothes that are too big Appears anxious when around people Has bruises around the wrists and forearms

Avoids eye contact Wears clothes that are too big Appears anxious when around people Has bruises around the wrists and forearms

A nurse is assessing the mouth of a client and finds that she has a smooth, red, shiny tongue without papillae. The nurse should recognize this as indicative of a loss of which vitamin? B12 C D K

B12

When assessing temperature of the skin, which portion of the hand should the examiner use? Fingertips Palms Backs of fingers Ulnar aspect of the hand

Backs of fingers

A client is being assessed following a motor vehicle accident. The client's right eye is swollen shut and very painful. Why does this require further assessment? Blunt-force trauma often results in fracture of the orbit High-velocity injuries are typically non-penetrating The client could have optic atrophy This could be a sign of strabismus

Blunt-force trauma often results in fracture of the orbit

Which of the following statements is true about biologic variation? Both genetics and environment produce biologic variation. Cultural practices produce biologic variation. Race is based on physical variations. Drug metabolism differences are not culture based.

Both genetics and environment produce biologic variation.

The nurse hears high-pitched swooshing sounds over the carotid artery on the right side. What is this sound indicative of? Bruits Murmurs Normal findings Gallops

Bruits

The nurse is palpating a client's cervical vertebrae. Which vertebra can be easily palpated when the neck is flexed and should help the nurse locate the other vertebrae? C1 C3 C5 C7

C7

The nurse is assessing a client who has been taking antibiotics for an infection for 10 days. The nurse observes whitish curd-like patches in the client's mouth. The nurse should explain to the client that these spots are most likely Candida albicans infection. Koplik spots. leukoplakia. Fordyce spots.

Candida albicans infection.

A client says that illness is a curse brought on by living outside the laws of God and is retribution for personal evil. Which religion is this client most likely practicing? Islam Judaism Buddhism Christianity

Christianity

A client presents to the health care clinic with reports of swelling, pain, and coolness of the lower extremities. The nurse should recognize that which of these lifestyle practices are risk factors for peripheral vascular disease? Select all that apply. Cigarette smoking Regular exercise Stress-reduction techniques Low alcohol intake Previous use of hormones High-fat diet

Cigarette smoking Previous use of hormones High-fat diet

The chambers of the eye contain aqueous humor, which helps to maintain intraocular pressure and transmit light rays. maintain the retinal vessels. change refractory of the lens. cleanse the cornea and the lens.

Cleanse the cornea and the lens

A nurse is unable to palpate the apical impulse on an older client. Which assessment data in the client's history should the nurse recognize as the reason for this finding? Client has an increased chest diameter Heart rate is irregular Respiratory rate is too fast Heart enlargement is present

Client has an increased chest diameter

What is responsible for the inspiratory splitting of S2? Closure of aortic then pulmonic valves Closure of mitral then tricuspid valves Closure of aortic then tricuspid valves Closure of mitral then pulmonic valves

Closure of aortic then pulmonic valves

During the admission assessment, the nurse identifies the client has a history of Raynaud's. What assessment finding would the nurse expect to find? Cold fingers and hands Cool legs bilaterally Cool leg on one side Capillary refill less than 2 seconds

Cold fingers and hands

The nurse is suspicious of a deep vein thrombosis (DVT) in a client's right calf area. Which actions should the nurse perform or assist with to aid in the diagnosis of DVT? (Select all that apply.) Assess for the Homan's sign Complete Wells Score System Draw lab work for D-dimer Prepare client for venous duplex scan Measure calf size in both legs

Complete Wells Score System Draw lab work for D-dimer Prepare client for venous duplex scan Measure calf size in both legs

The nurse is caring for a client who is from another country, and states to the charge nurse, "I just don't know if the client really understands what I am saying, and I am not understanding the client either, even though he speaks English." What stage of cultural awareness is the nurse experiencing? Unconscious incompetence Conscious incompetence Conscious competence Unconscious competence

Conscious incompetence

Which of the following assessment findings is most congruent with chronic arterial insufficiency? Brown pigmentation around a client's ankles and shins Ulceration on the medial surface of the client's ankle Thickened and scarred skin on the client's ankle Cool foot temperature and ulceration on the client's great toe

Cool foot temperature and ulceration on the client's great toe

An adult client tells the nurse that his eyes are painful because he left his contact lenses in too long the day before yesterday. The nurse should instruct the client that prolonged wearing of contact lenses can lead to retinal damage. cataracts. myopia. corneal damage.

Corneal damage

The nurse observes that a client who is Jewish and follows an Orthodox way of life. The client is not eating the food that the dietary department is sending. What would be the best lunch to bring this client? Cottage cheese, peaches, and crackers with tea. Roast beef sandwich, mashed potatoes, and a cup of ice cream. Sliced pork, stuffing, and green peas. Crab cakes, french fries, and a roll.

Cottage cheese, peaches, and crackers with tea.

A nurse is assessing a child who got lost on a camping trip in November and was exposed all night to the elements. Which finding about the lips would support a diagnosis of hypoxia in this client? Reddish Cyanotic Pallor Swelling

Cyanotic

During an interview with the nurse, a client complains of a fatigue that seems to get worse in the evening. Which of the following causes of fatigue would explain this pattern? Depression Severe muscular exertion Decreased cardiac output Upper respiratory infection

Decreased cardiac output

A nurse recognizes which finding as an indication of an ulcer due to arterial insufficiency? Deep ulcers that often involve joint space Ulcer commonly located in anterior tibial area Painful ulcer with irregular border Moderate to severe leg edema

Deep ulcers that often involve joint space

Many factors put older adults at risk for abuse. Which of the following are possible factors? Select all that apply. Dependency Cognitive decline Strained mental health of caregivers Decline in physical health of caregivers Geographical location

Dependency Cognitive decline Strained mental health of caregivers Decline in physical health of caregivers

A client just died, and the nurse is preparing the body for the funeral home. The client practiced Judaism and a rabbi was present at the time of death. The nurse is careful to do which of the following to honor the client's religious beliefs concerning death? Do not cross the client's arms. Burn all bandages with the client's blood. Discard all clothes of the client. Place a medal with the client.

Do not cross the client's arms.

A hospitalized client continues to exhibit residual effects of a stroke. Which symptom is the priority concern? Weak gait Dysphagia Right ptosis Facial weakness

Dysphagia

During assessment of the oral cavity, the nurse examines the salivary glands. Which area of the mouth should the nurse assess to inspect for the Wharton's ducts? Either side of the frenulum on the floor of the mouth Buccal mucosa across from the second upper molars Right side of the frenulum at the base of the gums Posterior aspect of the tongue bilaterally

Either side of the frenulum on the floor of the mouth

The nurse is discharging an adult client who received 18 staples for a head laceration received while mountain biking. What can the nurse focus on while doing discharge teaching? Encourage the use of safety equipment Encourage proper nutrition to promote healing Encourage the client to take a safety course Teach proper posture, bending, and lifting

Encourage the use of safety equipment

The nurse observes an inward turning of the lower lid in a 77-year-old client. The nurse documents entropion ectropion ptosis exophthalmos

Entropion

A nurse is educating a client about the function of the parts of the auditory system. Which is the function of the eustachian tube? Transmits vibration to the fluid filled inner ear at the oval window. Separates the external from the middle ear. Sends sensory information to the cerebellum and midbrain. Equalizes the pressure in the middle ear with atmospheric pressure.

Equalizes the pressure in the middle ear with atmospheric pressure.

A nurse recognizes that the belief that one's worldview is the only acceptable truth and that one's beliefs, values, and sanctioned behaviors are superior to all others is called what? Ethnocentrism Stereotyping Egocentrism Ethnicity

Ethnocentrism

What is a characteristic symptom of Graves hyperthyroidism? Pterygium Exophthalmos Pinguecula Episcleritis

Exophthalmos

Which of the following is an essential topic when discussing risk factors for peripheral arterial disease with a client? Exercise tolerance Prevention of varicose veins Extent of tobacco use and exposure Significance of cardiac dysrhythmias

Extent of tobacco use and exposure

A client diagnosed with Sjogren syndrome should be given which instructions? Eye drops and sucking on hard candy may used to relieve dryness. Blood pressure should be checked frequently. Condom use can reduce the risk of transmission. Taking mucus thinning medication can relieve symptoms.

Eye drops and sucking on hard candy may used to relieve dryness.

A nurse is using a quick reference to guide the spiritual assessment. A published acronym related to the assessment of spirituality is which of the following? FACES PIE FICA PRN

FICA

A nurse is assessing an elderly client who may be a victim of elder mistreatment. Which of following are examples of elder mistreatment? Select all that apply. Failure to provide adequate nutrition to an elder Shoving an elder into a wheelchair Administering the wrong medication to an elder inadvertently Forcing an elder to perform a sexual act Having an elder sign financial documents without an understanding of what is being signed

Failure to provide adequate nutrition to an elder Shoving an elder into a wheelchair Forcing an elder to perform a sexual act Having an elder sign financial documents without an understanding of what is being signed

Walking contracts the calf muscles and forces blood away from the heart. True False

False

A client exhibits purulent drainage in the right external ear canal. The client complains of pain that increases when the ear is touched. Which client teaching instructions should the nurse provide? Take over the counter common cold remedies to resolve the problem. Finish the entire course of antibiotic therapy. Permanent deafness is common with this condition. Tympanostomy tubes will usually fall out on their own over time.

Finish the entire course of antibiotic therapy.

A nurse receives an order to perform a compression test to assess the competence of the valves in a client's varicose veins. Which action by the nurse demonstrates the correct way to perform this test? Ask the client to sit on a chair for the examination Firmly compress the lower portion of the varicose vein Place the second hand 3 to 4 inches above the first hand Feel for a pulsation to the fingers in the lower hand

Firmly compress the lower portion of the varicose vein

A client visits a community clinic reporting severe allergies causing a "crackling sensation" in the ear. The physician diagnoses serous otitis media. Which of the following is a characteristic of this condition? Fluid collects in the middle ear causing an obstruction of the auditory tube. An upper respiratory infection spreads through the auditory tube. This condition develops if acute purulent otitis media is not treated promptly. This condition is usually associated with a puncture eardrum.

Fluid collects in the middle ear causing an obstruction of the auditory tube.

The student nurse is aware that culturally competent care is described as which of the following? Being aware of differences across groups but not recognizing that differences appear within the same group Following five constructs of cultural awareness, knowledge, skill, encounters, and desire Expecting that all members of a particular culture hold the same beliefs Realizing that the nurse's own culture is the best and all others are inferior

Following five constructs of cultural awareness, knowledge, skill, encounters, and desire

A client shares that a first-degree relative has an eye problem, but they not sure what the diagnosis is. What major eye problem will the nurse suggest screening the client for? Retinoblastoma Strabismus Retinitis pigmentosa Glaucoma

Glaucoma

An adult client tells the nurse that her peripheral vision is not what it used to be and she has a blind spot in her left eye. The nurse should refer the client for evaluation of possible glaucoma. increased intracranial pressure. bacterial infection. migraine headaches.

Glaucoma

A six-month old male infant is brought to the emergency department by his parents for inconsolable crying and pulling at his right ear. When assessing this infant the nurse is aware that the tympanic membrane should be what color in a healthy ear? Yellowish-white Red Gray Bluish-white

Gray

A nurse is interviewing a client who is a survivor of abuse. The client is telling the nurse about how the violence occurred. Which statement would the nurse interpret as reflecting phase 3 of the cycle of violence? "He threw me against the wall and starting punching my face." "He yells at me for not having dinner waiting for him when he came home." "He calls me stupid and incompetent, asking himself why he ever married me." 'He tells me that he is sorry and that he will never hit me again."

He tells me that he is sorry and that he will never hit me again

A nurse needs to assess a client who is experiencing chronic headache to determine how it is affecting her activities of daily living. Which of the following interventions should the nurse implement? A mnemonic assessment tool Headache Impact Test Auscultation Family health history questionnaire

Headache Impact Test

A nurse is preparing a community education session on hearing loss. Which information should the nurse include? Hearing loss can lead to mental health problems. All ethnic groups experience hearing loss in the same way. In order to hold a driver's license, hearing must be tested regularly. Hearing loss typically begins after the age of 40.

Hearing loss can lead to mental health problems.

During the health history interview with a 40-year-old man, the nurse uses the genogram to specifically assess for major family risk for cardiovascular disease by asking about which of the following? Hypertension in his grandparents Weight patterns within his family Diabetes mellitus in his extended family Heart attacks in his father and siblings

Heart attacks in his father and siblings

The eustachian tube is a passage between the middle ear and the nasopharynx. What is the function of the eustachian tube? Helps to regulate pressure in the middle ear Protects the middle ear Allows for drainage of fluid from the middle ear Maintains fluid in the middle ear

Helps to regulate pressure in the middle ear

Which finding should a nurse recognize as normal when assessing the ears of an elderly client? Decrease in cerumen production Shortened earlobes High-tone frequency loss Bulging tympanic membrane

High-tone frequency loss

You are caring for a client in the outpatient clinic with suspicion of cancer due to recent weight losses for unidentifiable reasons. The client a 25-year history of smoking. You perform an assessment and ask the client about symptoms related to laryngeal cancer. What is an early symptom associated with laryngeal cancer? Hoarseness Dyspnea Dysphagia Alopecia

Hoarseness

A staff educator from the hospital is providing an event for the hospital staff. The educator is talking about health promotion activities for people with diseases of the nose, mouth, throat, and sinuses. What would the educator include in the presentation? How to safely put an infant to bed with a bottle Beginning dental care in children at age 5 How to reduce periodontal disease How oral cancer is diagnosed

How to reduce periodontal disease

A nurse is working with a woman who has been abused by her husband. Which of the following is a physiological sign commonly associated with abuse? High blood sugar Decreased respiration rate Hypertension Decreased heart rate

Hypertension

A client presents at the clinic for a routine check-up. The nurse notes that she is dressed in warm clothing even though the temperature outside is 73°F (22.8°C). The nurse also notes that the client has gained 10 pounds (4.5 kg) since her last visit 9 months ago. What might the nurse suspect? Effects of age-related changes Brain tumor Hyperthyroidism Hypothyroidism

Hypothyroidism

A client diagnosed with goiter has undergone a thyroidectomy. Which statement from the client indicates understanding of post-operative care teaching? I must take thyroid hormone replacement medication for the rest of my life. I will complete the entire course of thyroid hormone replacement over six weeks. I must keep my follow up appointments to receive my thyroid hormone injections. I will take my thyroid hormone replacement medication once every week.

I must take thyroid hormone replacement medication for the rest of my life.

The nursing instructor is educating her students on the important of assessing for victims of abuse and violence. What statement by the students indicate an understanding of when to assess for abuse and violence? "I will assess a client for abuse and violence only when they present with bruises or a broken limb." "I will assess a client for abuse and violence with every client encounter." "It is not necessary to assess every client. Some clients get offended when you ask them those questions." "I will assess pregnant clients who come to the clinic."

I will assess a client for abuse and violence with every client encounter

Impaired dilation of the eye is evaluated with an assessment of which cranial nerve (CN)? II (optic) III (oculomotor) IV (trochlear) VI (abducens)

III (oculomotor)

When caring for a client from a culturally different background, what is the goal for incorporating the client's health beliefs and practices into the nursing plan of care? To enhance the client's social system To enhance cultural connectedness Improvement of the client's health outcomes Improvement of communication with the client and family

Improvement of the client's health outcomes

The nurse assesses a client's submental lymph nodes. In which area of the client's head should the nurse palpate these lymph nodes? Superficial to the sternomastoid In front of the ear In the midline, a few centimeters behind the tip of the mandible At the angle of the mandible

In the midline, a few centimeters behind the tip of the mandible

The nurse understands that when the sympathetic nervous system is stimulated what occurs? Select all that apply. Increased cardiac output Decreased cardiac output Increased blood pressure Decreased blood pressure Increased heart rate

Increased cardiac output Increased blood pressure Increased heart rate

A client is admitted to the health care facility with reports of chest pain, elevated blood pressure, and shortness of breath with activity. The nurse palpates the carotid arteries as 1+ bilaterally and a weak radial pulse. A Grade 3 systolic murmur is auscultated. Which nursing diagnosis can the nurse confirm based on this data? Impaired Breathing Pattern Activity Intolerance Ineffective Health Maintenance Ineffective Tissue Perfusion

Ineffective Tissue Perfusion

A nurse cares for a client who suffered a myocardial infarction 2 days ago. A high-pitched, scratchy, scraping sound is heard that increases with exhalation and when the client leans forward. The nurse recognizes this sound as a result of what process occurring within the pericardium? Increased pressure within the ventricles Inability of the atria to contract Inflammation of the pericardial sac Incompetent mitral valve

Inflammation of the pericardial sac

A middle-aged client reports difficulty in reading. Which action by the nurse is appropriate to test near visual acuity using a Jaeger reading card? Place the chart 20 feet away from the client on the wall Instruct the client to hold the chart away from the body at arm's length Instruct the client hold the chart 14 inches from the eyes Place the chart on a table 17 inches away from the client

Instruct the client hold the chart 14 inches from the eyes

Which of the following are forms of psychological (emotional) abuse? Select all that apply. Insulting Humiliating Threatening to destroy property Rape Incest

Insulting humiliating threatening to destroy property

A 57-year-old maintenance worker comes to the office for evaluation of pain in his legs. He is a two-pack per day smoker since the age of 16, but he is otherwise healthy. The nurse is concerned that the client may have peripheral arterial disease. Which of the following is a common symptom that could indicate peripheral arterial disease? Intermittent claudication Chest pressure with exertion Shortness of breath Knee pain

Intermittent claudication

Which is true of a third heart sound (S3)? It marks atrial contraction. It reflects normal compliance of the left ventricle. It is caused by rapid deceleration of blood against the ventricular wall. It is not heard in atrial fibrillation.

It is caused by rapid deceleration of blood against the ventricular wall.

The nurse wants to support a client's spirituality. To do so, what must the nurse do first? Give the client a Bible to read. Keep an objective perspective and meet the client at his or her level. Ask the on-call chaplain to come see the client. Tell the client that religion is important to you and you are happy to discuss it.

Keep an objective perspective and meet the client at his or her level.

An emergency department nurse is caring for a young child with intractable nose bleeds. What is the most common site of epistaxis? Kiesselbach plexus Rosenmuller fossa Columella Ala

Kiesselbach plexus

During an oral assessment, the nurse identifies that client has white patches in his mouth. How would this be documented in the medical record? Leukoplakia Petechiae Gingivitis Fordyce granules

Leukoplakia

During your physical examination of the client you note an enlarged tender tonsillar lymph node. What would you do? Assess for meningitis Look for involvement of other regions of the body Look for a source such as infection in the area that it drains Assess for dietary changes

Look for a source such as infection in the area that it drains

After completing a spiritual assessment, a nurse determines that the client has signs and symptoms requiring medical diagnosis and treatment. What should the nurse do? Return to the client to further discuss spirituality. Make a referral to a primary care provider. Treat the medical condition. Ignore the medical problem.

Make a referral to a primary care provider.

An older adult client who wears dentures reports having soreness of the gums. Which intervention should the nurse recommend to the client to alleviate this problem? Avoid excessive intake of sugary foods. Use toothpaste containing fluoride. Have a dental examination every 2 years. Massage the gums daily.

Massage the gums daily.

Sickle cell disease and other hemoglobinopathies such as thalassemia are often found in persons originating from which geographical regions? Central and South America. Northern and Central Europe. Mediterranean and Africa. Australia and New Zealand.

Mediterranean and Africa.

A client reports right-sided temporal headache accompanied by nausea and vomiting. A nurse recognizes that which condition is likely to produce these symptoms? Bell's palsy Tension headache Temporal arteritis Migraine headache

Migraine headache

A nurse is performing a head and neck assessment on a client. Which area should the nurse inspect for facial symmetry? Nasolabial folds Temporomandibular joint Preauricular nodes Earlobes

Nasolabial folds

When assessing the tympanic membrane, where would the nurse expect to visualize the malleus? In the anterior aspect In the area of the cone of light Near the center To the left of the cone of light

Near the center

A nurse is assessing a boy who appears to be undernourished. She can find no signs of physical abuse. She considers whether this may be a case of child abuse. Which of the following are included in the definition of child abuse? Select all that apply. Neglect Physical abuse Sexual abuse Intimate partner violence Elder abuse Emotional abuse

Neglect Physical abuse Sexual abuse Emotional abuse

The nurse notes that an adolescent male has ptosis of the left eye. What should the nurse suspect as the reason for this finding? Undiagnosed eye disease Need for corrective lenses Undiagnosed neurologic disease Nerve damage caused by repeated eye injuries

Nerve damage caused by repeated eye injuries

When teaching a class of school-age children about hygiene, the nurse should include which information about the ears? Ears should be cleaned with cotton applicators once a day at bedtime. Never put anything smaller than your elbow in your ears. Only allow your parents clean out your ears with cotton applicators. Producing less earwax can lead to a hearing loss.

Never put anything smaller than your elbow in your ears.

If palpable, superficial inguinal nodes are expected to be: Fixed, tender, and at 2.5 cm in diameter Discrete, tender, and 2 cm in diameter Nontender, mobile, and 1 cm in diameter Fixed, nontender, and 1.5 cm in diameter

Nontender, mobile, and 1 cm in diameter

A mother brought a child in to the Emergency Department stating that she thinks her child's appendix has ruptured. Before any diagnostic tests can be done, the father comes in and says, "I don't want anything done, we will take the child to our church where prayer will heal him." What is an appropriate action by the nurse at this time? Notify the ethics committee immediately. Tell the father that if he takes the child from the Emergency Department, the police will be notified. Lock the father out of the Emergency Department. Tell the mother and father that you will call social service if they do not allow treatment.

Notify the ethics committee immediately

The nurse palpates a client's auricles and notes an enlarged lymph node on one ear. No redness is observed, and the client denies pain or tenderness. What is the nurse's best action? Document the finding as an isolated benign node. Inform the client of the need for ear drops. Refer the client to an audiologist for an audiogram. Notify the healthcare provider about the finding.

Notify the healthcare provider about the finding.

A client complains of a unilateral headache near the scalp line and double vision. The nurse palpates the space above the cheekbone near the scalp line on the affected side, and the client complains of tenderness on palpation. What is the nurse's next action? Notify the healthcare provider immediately. Administer intravenous pain medication. Palpate the carotid pulses bilaterally at the same time. Prepare the client for a temporal artery biopsy.

Notify the healthcare provider immediately.

A hospitalized post-operative client exhibits edema, pain, erythema, and warmth in the right calf area. What is the nurse's best action? Encourage early ambulation. Assist the client to turn, cough, and deep breathe. Discontinue the indwelling urinary catheter. Notify the healthcare provider.

Notify the healthcare provider.

The nurse understands that the best plans for referral or intervention will develop from what part of the nurse-client interaction? Objective assessment Ongoing dialogue Asking specific questions. Using silence to encourage the client to talk.

Ongoing dialogue

A 58-year-old teacher presents with breathlessness with activity. The client has no chronic conditions and does not take any medications, herbs, or supplements. Which of the following symptoms is appropriate to ask about in the cardiovascular review of systems? Abdominal pain Orthopnea Hematochezia Tenesmus

Orthopnea

A client complains of difficulty sleeping, stating he has to sit up with the help of several pillows and cannot breathe when lying flat. This client has a condition known as what? Orthopnea Tachypnea Pneumonia Sleep apnea

Orthopnea

A client with hearing loss by whisper test is further examined with a tuning fork, using the Weber and Rinne maneuvers. The abnormal results are as follows: bone conduction is greater than air on the left, and the client hears the tuning fork better on the left. Which of the following is most likely? Otosclerosis of the left ear Exposure to chronic loud noise of the right ear Otitis media of the right ear Perforation of the right eardrum

Otosclerosis of the left ear

The community health nurse is reviewing recommendations from the US Preventive Services Task Force (USPSTF) to help identify risk factors for intimate partner violence. On which community factors should the nurse focus? Select all that apply. Overcrowding Marital instability Lack of institutions Strict gender norms Neighbor unwillingness to intervene

Overcrowding Lack of institutions Neighbor unwillingness to intervene

The nurse is conducting a workshop on the measurement of jugular venous pulsation. As part of instruction, the nurse tells the students to make sure that they can distinguish between the jugular venous pulsation and carotid pulse. Which of the following characteristics is typical of the carotid pulse? Palpable Soft, rapid, undulating quality Pulsation eliminated by light pressure on the vessel Level of pulsation changes with changes in position

Palpable

As a novice nurse caring for a client from a different culture, what may the nurse find confusing or upsetting? Patient's level of motivation Patient's nonverbal communication Patient's family members Patient's ethnicity

Patient's nonverbal communication

A client complains of recurring headaches that are worse when first waking in the morning and with coughing or sneezing. What would be the nurse's most appropriate action? Ask the doctor for an order for an MRI Perform a focused assessment Prepare the client for a spinal tap Perform a generalized assessment

Perform a focused assessment

What are some basic rules for nurses to follow when assessing for violence? Select all that apply. Perform assessment and screening only when the client is alone in a safe, private environment Be very patient when the client talks Demonstrate compassion, not judgment Ask only specific questions Make sure a family member is present

Perform assessment and screening only when the client is alone in a safe, private environment Be very patient when the client talks Demonstrate compassion, not judgment

What is the most important physical sign of acute pericarditis? Pericardial friction rub Intense pain Elevated white cell count Murmur heard over the left sternal border

Pericardial friction rub

A client complains of pain in the calves, thighs, and buttocks whenever he climbs more than a flight of stairs. This pain, however, is quickly relieved as soon as he sits down and rests. The nurse should suspect which of the following conditions in this client? Advanced chronic arterial occlusive disease Neuropathy secondary to diabetes Venous disease Peripheral arterial disease

Peripheral arterial disease

What pulse is located in the groove between the medial malleolus and the Achilles tendon? Posterior tibial Dorsalis pedis Popliteal Femoral

Posterior tibial

A nurse notices a middle-aged client in the waiting room pick up a magazine to read while she waits to be seen. She opens the magazine and then extends her arms to move it further from her eyes. Which condition does the nurse most suspect in this client? Exotropia Esotropia Strabismus Presbyopia

Presbyopia

A client reports severe pain in the posterior region of the neck and difficulty turning the head to the right. What additional information should the nurse collect? Previous injuries to the head and neck Difficulty with swallowing Changes in sleeping habits Stiffness in the right shoulder

Previous injuries to the head and neck

When testing the near reaction, an expected finding includes which of the following? Pupillary dilation on near gaze; dilation on distant gaze Pupillary dilation on near gaze; constriction on distant gaze Pupillary constriction on near gaze; dilation on distant gaze Pupillary constriction on near gaze; constriction on distant gaze

Pupillary constriction on near gaze; dilation on distant gaze

A nurse documents on the client's assessment form, "skin color appropriate for race". Why is this not an appropriate assessment statement? Race is not a physical characteristic. The nurse needs to be culturally sensitive when making comments such as these. This is ethnocentrism and should not be documented. It is appropriate to use this statement because people of the same race all have the same skin color.

Race is not a physical characteristic.

While discussing care needs a client states the desire to be positive about the upcoming medical treatments and expect them to help cure the disease. Which nursing diagnosis should the nurse identify for this client? Hopelessness Spiritual distress Risk for spiritual distress Readiness for enhanced hope

Readiness for enhanced hope

ile discussing care needs a client states the desire to be positive about the upcoming medical treatments and expect them to help cure the disease. Which nursing diagnosis should the nurse identify for this client? Hopelessness Spiritual distress Risk for spiritual distress Readiness for enhanced hope

Readiness for enhanced hope

After completing an admission assessment the client asks if the room door can be closed so that the client can have silence when reading the Bible. What does this indicate to the nurse about the client's spirituality? The client has limited reading material The client reads the Bible instead of praying Reading the Bible provides comfort to the client Noise interrupts the client's ability to concentrate when reading

Reading the Bible provides comfort to the client

Which characteristic feature of the tympanic membrane should a nurse anticipate finding in a client with acute otitis media? Pearly, translucent, with no bulging Yellowish, bulging, with fluid bubbles Gray, translucent, with no retraction Red, bulging, with an absent light reflex

Red, bulging, with an absent light reflex

A nurse is examining the nose of a client diagnosed with an upper respiratory tract infection. Which characteristics of the nasal mucosa should the nurse expect to find during assessment of a client with an upper respiratory tract infection? Dark pink, moist, & free of discharge Pale pink, swollen, with watery exudate Bluish gray, swollen, with watery exudate Red, swollen, with purulent discharge

Red, swollen, with purulent discharge

A client reports using pain medication and sitting in a dark room on the onset of a migraine headache. In which part of the subjective section of the physical examination should the nurse document this information? onset location treatment relieving factors

Relieving factors

A client asks the nurse if there are church services in the hospital because the client attends mass every Sunday. The nurse realizes that this client is demonstrating: Religion Culture Spirituality Recreation

Religion

When interviewing a pediatric client and attempting to determine the presence of abuse, the nurse should confine the interview to yes/no questions to keep the interview simple. remain calm and accepting in response to any information the client discloses. ask leading questions to convince the child to offer information. offer a reward to the child for answering difficult questions.

Remain calm and accepting in response to any information they client discloses

A client who is taking antibiotics for a sinus infection presents with a white coating on the tongue and complains of a burning sensation on the tongue. Which instructions are most appropriate for this client? Scrape off the white coating with a toothbrush. Clean mouth and tongue with hydrogen peroxide and water. Rinse mouth with antifungal medication as prescribed. Apply the prescribed topical antibiotic gel as directed.

Rinse mouth with antifungal medication as prescribed.

A trauma client reports pain in the left lower extremity. The nurse notes that the extremity has pallor. Pedal pulses are diminished, and paresthesia is present. What nursing diagnosis might the nurse use? Pain related to decreased blood flow and altered tissue perfusion Activity intolerance related to pain and claudication with ambulation Altered tissue perfusion, arterial related to reduced blood flow Risk for peripheral neurovascular dysfunction

Risk for peripheral neurovascular dysfunction

When auscultating the heart, the nurse is most likely to hear a diastolic murmur after which heart sound? S1 S2 Preload Afterload

S2

After reviewing a client's completed danger assessment questionnaire, the nurse determines that the client is in significant danger of intimate partner violence (IPV) leading to homicide. The client says that she would prefer to return home and that she does not have a safety plan. Which of the following nursing interventions should the nurse implement at this point? Select all that apply. Encourage the client to return home to avoid raising suspicion in the abuser. Schedule a follow-up appointment. Have the client complete Assessment Tool 10-2: Assessing a Safety Plan. Provide the client with contact information for shelters and groups. Encourage the client to call with any concerns. Call the police and ask them to pick up the abuser.

Schedule a follow-up appointment. Have the client complete Assessment Tool 10-2: Assessing a Safety Plan. Provide the client with contact information for shelters and groups. Encourage the client to call with any concerns.

A client is assigned a visual acuity of 20/100 in her left eye. Which of the following is true? She obtains a 20% correct score at 100 feet. She can accurately name 20% of the letters at 20 feet. She can see at 20 feet what a normal person could see at 100 feet. She can see at 100 feet what a normal person could see at 20 feet.

She can see at 20 feet what a normal person could see at 100 feet.

A client comes to the emergency department reporting a sudden onset of dyspnea. What finding is a manifestation of dyspnea? Shortness of breath Painful breathing Rapid breathing Inability to breathe

Shortness of breath

When performing a cultural assessment on a client from another country, what can a nurse do to ensure the client receives culturally sensitive care? Show genuine interest in the client's culture and personal life experiences. Develop a plan of care after talking to the client's family. Develop a plan of care based on the nurse's own knowledge of a client's culture. After the focused assessment, decide if a cultural assessment is necessary.

Show genuine interest in the client's culture and personal life experiences.

A nurse auscultates the heart rate of a young male and notices that the rate speeds with inspiration and slows with exhalation. S1 and S2 are normal. The nurse recognizes this as what dysrhythmia? Premature ventricular contractions Atrial fibrillation Sinus arrhythmia Premature atrial contractions

Sinus arrhythmia

The nurse is providing teaching about cardiovascular disease in a community setting. What risk factors would the nurse identify to the group as those they can modify through lifestyle choices? Select all that apply. Smoking Blood pressure Cholesterol Family history Age

Smoking Blood pressure Cholesterol

Goals, although not specific for peripheral vascular disease, focus on areas of risk. What are these areas of modifiable risk? Select all that apply. Smoking Overweight Lack of exercise Family history Ethnicity

Smoking Overweight Lack of exercise

During an interview, a client tells the nurse about spending time out of doors and hiking to connect with the earth and observe the activities of small animals. The nurse would document this client's hiking behavior as supporting: Exercise needs Religious activities Spirituality Relaxation

Spirituality

What systemic diseases may cause nodular episcleritis? (Mark all that apply.) Systemic lupus erythematosus Multiple sclerosis Fibromyalgia Muscular dystrophy Rheumatoid arthritis

Systemic lupus erythematosus Rheumatoid arthritis

As part of a physical assessment, the nurse performs the confrontation test to assess the client's peripheral vision. Which test result should a nurse recognize as indicating normal peripheral vision for a client using the confrontation test? Client's consensual pupil constricts in response to indirect light. Eyes converge on an object as it is moved towards the nose. Direct light shown into the client's pupils results in constriction. The client and the examiner see the examiner's finger at the same time.

The client and the examiner see the examiner's finger at the same time.

The nurse observes the spouse of a client pinch the client's arm when someone talks with the client in the waiting room of the community clinic. What should this observation indicate to the nurse? The spouse is jealous The spouse was trying to get the client's attention The client is not permitted to have contact with others The client was sharing too much information with the other person

The client is not permitted to have contact with others

The nurse is setting an outcome with the client experiencing spiritual distress. Which goal would be appropriate? The client will express meaning and purpose in life. The client will attend a community event every week. The client will express feelings of connectedness with others. The client will initiate interaction with others.

The client will express meaning and purpose in life.

What outcome should the nurse prioritize when addressing a client's social, cultural, and spiritual issues? The client will express meaning and purpose in life. The client will engage in interactions with others. The client will express a sense of oneness with self and others. The client will engage in worship weekly.

The client will express meaning and purpose in life.

Where are the heart and great vessels located in the human body? The mediastinum, between the lungs above the diaphragm The mediastinum, between the lungs below the diaphragm The peritoneum, above the diaphragm The peritoneum, below the diaphragm

The mediastinum, between the lungs above the diaphragm

The nurse is caring for a first-time mother and newborn in the postpartum unit. The nurse overhears the new mother ask family members to prepare the nursery and purchase clothing for the baby. What would the culturally sensitive nurse suspect? The new mother may believe buying infant clothing before the delivery is bad luck. The new mother is young, and the pregnancy was unexpected and the baby is unwanted. The new mother and requires education about how to prepare for the new baby. The mother is from a lower socioeconomic level and is not able to buy what is needed for the baby.

The new mother may believe buying infant clothing before the delivery is bad luck

Which action by the nurse is consistent with Weber's test? The nurse activates the tuning fork and places it on the midline of the parietal bone in line with both ears. The nurse strikes the tuning fork and places it on the client's mastoid process to measure bone conduction. The nurse uses a bulb insufflator attached to an otoscope to observe movement of the tympanic membrane. The nurse shields their mouth and whispers a simple sentence approximately 18 inches from the client's ear.

The nurse activates the tuning fork and places it on the midline of the parietal bone in line with both ears.

What must occur before a client will be willing to teach the nurse about her culture? The client's family must allow it The nurse needs to hide her biases The nurse needs to establish rapport and trust The nurse needs to establish a value-based partnership

The nurse needs to establish rapport and trust

Which action by the nurse is consistent with the Rinne test? The nurse strikes the tuning fork and places it on the client's mastoid process to measure bone conduction. The nurse activates the tuning fork and places it on the midline of the parietal bone in line with both ears. The nurse uses a bulb insufflator attached to an otoscope to observe movement of the tympanic membrane. The nurse shields their mouth and whispers a simple sentence approximately 18 inches from the client's ear.

The nurse strikes the tuning fork and places it on the client's mastoid process to measure bone conduction.

The nurse prepares to complete a spiritual assessment with a client. What should the nurse keep in mind when completing this assessment? It offers the nurse a chance to subjectively discuss the client's spiritual beliefs The reason is to better understand the client's spiritual perspective related to health The reason is for the nurse to share personal views in relation to what the client believes It gives the nurse an opportunity to speak freely about personal spiritual beliefs and practices

The reason is to better understand the client's spiritual perspective related to health

Palpation of a 15-year-old boy's submandibular lymph nodes reveals them to be enlarged and tender. What is the nurse's most reasonable interpretation of this assessment finding? The boy requires assessment of his thyroid gland. There is an inflammatory response in the musculature of the boy's neck. The tissue underlying the nodes is infected. There is an infection in the area that these nodes drain.

There is an infection in the area that these nodes drain.

While performing a routine check-up on an 81-year-old retired grain farmer in the vascular surgery clinic, the nurse notes that he has a history of chronic arterial insufficiency. Which of the following physical examination findings of the lower extremities would be expected with this disease? Normal pulsation Normal temperature Marked edema Thin, shiny, atrophic skin

Thin, shiny, atrophic skin

A 58-year-old man who is HIV-positive has presented with thick, white plaques on his oral mucosa. What diagnosis would the nurse first suspect? Diphtheria Kaposi's sarcoma Torus palatinus Thrush

Thrush

During a pharmacology class the students are told that some drugs need to be closely monitored. What aspect should the nurse closely monitor for in clients who have been administered salicylates, loop diuretics, quinidine, quinine, or aminoglycosides? Signs of hypotension Reduced urinary output Tinnitus and sensorineural hearing loss Impaired facial movement

Tinnitus and sensorineural hearing loss

A young man is concerned about a hard mass in the midline of his palate that he has just noticed. Examination reveals that it is indeed hard and in the midline. No mucosal abnormalities are associated with this lesion. The client has no other symptoms. What is the most likely diagnosis? Leukoplakia Torus palatinus Thrush (candidiasis) Kaposi's sarcoma

Torus palatinus

How can a nurse accurately assess the distant visual acuity of a client who is non-English speaking? Move an object through the six cardinal positions of gaze Use a Snellen E chart to perform the examination Have the client read from a Jaeger reading card Perform the confrontation test

Use a Snellen E chart to perform the examination

Which food is most appropriate for the nurse to recommend for a client who suffers frequent nosebleeds due to hereditary hemorrhagic telangiectasia? Vegetable omelet Garlic chicken Chocolate pudding Salad with ginger dressing

Vegetable omelet

When you enter the room of a hospitalized client, you note that the client is guarding her left leg, which is swollen and reddened. You should identify the signs and symptoms of what complication of hospitalization? Decreased mobility Sepsis Venous thromboembolism Fluid imbalance

Venous thromboembolism

When visualizing the structures of the nose, the nurse recalls that air travels from the anterior nares to the trachea through the: Ala nasi, turbinates, and nasopharynx Ala nasi, vestibule, and ethmoid sinuses Vestibule, nasal passages, and nasopharynx Turbinates, ethmoid sinuses, and nasal passages

Vestibule, nasal passages, and nasopharynx

A nurse provides prevention strategies to a group of clients who are identified as at risk for hypertension. Which strategies should the nurse include? Select all that apply. Walk briskly 30 minutes per day. Use a low sodium seasoning to flavor food. Choose foods like bananas and sweet potatoes. Consume two to three glasses of red wine daily. Increase consumption of dairy products.

Walk briskly 30 minutes per day. Use a low sodium seasoning to flavor food. Choose foods like bananas and sweet potatoes.

A nurse performs an initial examination of a client brought to the emergency department after sustaining a head injury in an automobile accident. Which characteristic of discharge from the ears should alert the nurse that the client has a cerebrospinal fluid leak? Sticky or yellow Bloody & purulent Brown & odorless Watery or bloody

Water or bloody

The nurse suspects that a female client is the victim of abuse. What physical assessment finding caused the nurse to come to this conclusion? Select all that apply. Welts across the back Bruises around the wrists Bruising around the left eye and neck Tan lines around the chest and lower abdomen Healing circular wounds along the inner thighs

Welts across the back Bruises around the wrists Bruising around the left eye and neck Healing circular wounds along the inner thighs

A Muslim teenager is in the hospital for surgical repair of a severe fracture in the leg during the month of Ramadan, which the client says he would like to observe during his stay. Which of the following interventions should the nurse be prepared for in caring for this client? Withhold all food and drink between sunrise and sunset Serve meals as normal, but do not include pork Serve only kosher meats at all three meals Do not mix dairy and meats together on the client's tray

Withhold all food and drink between sunrise and sunset

Spirituality can best be described as a part of community identification. necessary for healing. a source of inner strength. another way of expressing a connection to a church.

a source of inner strength.

The functional reflex that allows the eyes to focus on near objects is termed pupillary reflex. accommodation. refraction. indirect reflex.

accommodation

While assessing an adult client's skull, the nurse observes that the client's skull and facial bones are larger and thicker than usual. The nurse should assess the client for parotid gland enlargement. acromegaly. Paget disease. Cushing syndrome.

acromegaly.

A client reports experiencing chronic headache after a recent upper respiratory tract infection. On physical examination, the nurse notes tenderness when palpating over the sinuses. Which condition is likely? acute bacterial sinusitis allergic rhinitis rhinitis medicamentosa epistaxis

acute bacterial sinusitis

After examining the client's tympanic membranes, the nurse documents "Right tympanic membrane, red and bulging with no light reflex." The nurse recognizes that these are signs of acute otitis media. serous otitis media. skull trauma. trauma from infection.

acute otitis media.

The meibomian glands secrete an oily substance to lubricate the eyes. sweat. hormones. clear liquid tears.

an oily substance to lubricate the eyes.

The nurse suspects a child is a victim of abuse. What observation caused the nurse to make this clinical determination? Hair needs to be combed Has a small hole in the shirt Wears shorts and tennis shoes Appears younger than stated age

appears younger than stated age

The nurse is inspecting the cornea and lens of an elderly client and notices a white arc around the limbus of the client's eye. The nurse recognizes this condition, common in older adults, as which of the following? Arcus senilis Presbyopia Ectropion Myopia

arcus senilis

A nurse is assigned to care for a client who has been physically abused by her husband. The nurse finds that client has an abuse score of 4 in her documents. Which of the following descriptions corresponds to the abuse score? Beating up and severe contusions Punching and kicking Head injury and internal injury Threat of abuse by weapons

beating up and severe contusions

The bicuspid, or mitral, valve is located between the left atrium and the left ventricle. between the right atrium and the right ventricle. at the beginning of the ascending aorta. at the exit of each ventricle near the great vessels.

between the left atrium and the left ventricle.

A nurse assesses a 4-year-old boy who experiences gas, a stomachache, and diarrhea after consuming lactose products. The nurse recognizes the data as what type of biological variation? Anatomic Biochemical Developmental Body surface

biochemical

The client comes to the health care provider stating he has a sore throat and believes he needs an antibiotic. This is an example of what type of model for health? Biomedical Complementary Spiritual Alternative

biomedical

In preparing to assess a client from a different culture, a nurse is aware that one needs to assess both factors that affect the client's approach to providers and factors that affect the client's disease, illness, and health state. Which of the following is a factor the nurse should consider that primarily affects the client's approach to providers? Death rituals Body language Pain Pregnancy and childbearing

body language

After palpating the radial pulse of an adult client, the nurse suspects arterial insufficiency. The nurse should next assess the client's femoral pulse. popliteal pulse. brachial pulse. tibial pulse.

brachial pulse.

The nurse is assessing a 59-year-old gas station owner for atherosclerosis in the lower extremities. In which of the following locations would the client's pain be most concerning? Thigh Knee Calf Ankle

calf

A nurse asks the client to describe the pain associated with a headache by rating the pain on a scale from 1 to 10. This subjective data should be documented in which section of the assessment? characteristic symptoms associated manifestations relieving factors location

characteristic symptoms

When describing the cycle of violence to a community group, the nurse explains that the first phase usually is: Somehow triggered by the victim's behavior Characterized by tension-building and minor battery Associated with loss of physical and emotional control Like a honeymoon that lulls the victim

characterized by tension-building and minor battery

A male college student presents to the student health clinic with reports of night-time headaches for the past 2 weeks. He denies nausea or photosensitivity but states that he has noticed his eyes are tearing and his nose runs a lot. He is stressed because of final exams and confesses to drinking more alcohol than normal. The nurse recognizes these findings as indicative of what type of headache? Migraine Cluster Tension Stress

cluster

A nurse is examining a 16-year-old girl who is visibly distraught. The client has a bruise on her face and tells the nurse that her boyfriend got rough with her recently. On further questioning, the client tells the nurse that her boyfriend raped her. Which of the following is the priority nursing intervention at this point? Apply ice to the bruise on the client's face to reduce swelling Conduct a forensic interview Assess the client for signs of psychological abuse Determine whether the boyfriend was abused as a child

conduct a forensic interview

A nurse is examining the eyes of a client who has complained of having a feeling of a foreign body in his eye. The nurse examines the thin, transparent, continuous membrane that lines the inside of the eyelids and covers most of the anterior eye. The nurse recognizes this membrane as which of the following? Retina Sclera Cornea Conjunctiva

conjunctive

A light is pointed at a client's pupil, which then contracts. It is also noted that the other pupil contracts as well, though it is not exposed to bright light. Which of the following terms describes this latter phenomenon? Direct reaction Consensual reaction Near reaction Accommodation

consensual reaction

A nurse is inspecting the ears of an Asian client and observes that her earlobes appear soldered, or tightly attached to adjacent skin with no apparent lobe. Which of the following should the nurse do next? Notify the physician of the finding Ask the client whether she has ever experienced an injury involving her ears Continue with the examination Record the finding and plan to follow-up at the client's next visit to note any changes

continue with the examination

A client has been diagnosed with astigmatism. The nurse should be prepared to teach the client about which treatment for this condition? Surgery Daily use of eye drops Corrective lenses No night driving

corrective lenses

A new graduate nurse from a small community college is going to work in an urban setting. She embraces the thought of working in a more diverse setting and wants to learn about different cultures. What behavior is this nurse exhibiting? Cultural desire Cultural knowledge Cultural skill Conscious incompetence

cultural desire

A male nurse is caring for an elderly woman who has become withdrawn and somewhat confused since the nurse has come on duty. When the nurse goes into the room to bathe the client, she refuses to allow it. The best explanation for her actions would be which of the following? confusion depression fear in strange surroundings cultural differences

cultural differences

The nurse knows that she will be caring for a client from Southern Korea who has been visiting with a family member. The nurse obtains some information about the Korean culture so that she will be able to meet the clients needs. What is the nurse demonstrating? Cultural knowledge Cultural assessment Unconscious incompetence Unconscious competence

cultural knowledge

The diagnosis of superficial phlebitis increases the client's risk for which vascular disorder? deep vein thrombosis compartment syndrome acute lymphangitis acute cellulitis

deep vein thrombosis

The nurse is caring for a woman in the prenatal clinic who comments that she just cannot seem to get things "right" anymore at home and that her husband says she knows so little about life. Which type of abusive or controlling behavior is the woman describing? Emotional abuse Minimizing Intimidation Blaming

emotional abuse

The spouse of a client believed to be a victim of intimate partner violence refuses to leave the room for the nurse to complete an assessment. What should the nurse do first? Call the police Ensure for personal safety Keep the room door closed Ask Security to remove the spouse

ensure for personal safety

Which technique should a nurse implement when interviewing with a child who has suffered physical abuse by the father? Establish a reassuring environment Offer rewards to the child for answering Ask questions that can be answered as "yes" or "no" Ask detailed questions with a lot of information

establish a reassuring environment

A 29-year-old physical therapist presents for evaluation of an eyelid problem. On observation, the right eyeball appears to be protruding forward. Based on this description, what is the most likely diagnosis? Ptosis Exophthalmos Ectropion Epicanthus

exophthalmos

When performing the cover test, a nurse notices that the client's left eye turns outward. How should the nurse document this finding in the client's record? Exotropia Esotropia Strabismus Presbyopia

exotropia

The nurse has established a diagnosis of "Readiness for enhanced spiritual well-being" for a client. Some client outcomes should include which of the following? (Select all that apply.) expresses hope expresses anger expresses acceptance describes ways to cope

expresses hope expresses acceptance

A nurse is aware that religion can sometimes negatively affect health. Which of the following is an example? failure to seek timely and proper medical care receiving blood transfusions taking multiple medications practicing yoga

failure to seek timely and proper medical care

The most useful spiritual assessment techniques should have specific introductory questions and be specific to a particular religious denomination to guide precise questions. True False

false

When the nurse tells the client that she will return in 1 hour to administer an intravenous antibiotic, the nurse is showing that she values time in which of the following orientations? future past present none of the above

future

During the physical examination of the mouth, the nurse identifies vesicular eruptions along the client's lips and surrounding skin. The nurse should document which problem? angular cheilitis herpes simplex actinic cheilitis angioedema

herpes simplex

A nurse is working with a client who confides in the nurse that her boyfriend will not let her get a job and insists on controlling all of their money. He gives her only a small allowance each month to buy food and a few necessities. When the nurse offers information on how to seek help for economic abuse, the client just shakes her head and says, "It's no use. Nothing can be done." Which of the following nursing diagnoses would be most appropriate in this situation? Anxiety related to not having enough money to purchase necessities due to significant other Dysfunctional family processes related to family violence Risk for violence (other directed) related to taking out frustrations with an abusive partner on their children Hopelessness related to remaining in a prolonged abusive relationship and inability to seek counseling and healthy supportive relationships

hopelessness related to remaining in a prolonged abusive relationship and inability to seek counseling and healthy supportive relationships

The nurses assesses the thyroid gland of a client with recent weight loss. On auscultation, a low, soft, rushing sound is heard over the lateral lobes. Which condition is most likely? hyperthyroidism thyroid cyst Hashimoto thyroiditis benign tumor

hyperthyroidism

Because the nurse realizes that spirituality varies, information gained will assist the nurse in individualizing interventions to meet specific needs. diagnosing the client with spiritual distress. teaching strict adherence to rituals and practices to improve outcomes. providing an overview of widely held beliefs from the major religions.

individualizing interventions to meet specific needs.

What nursing diagnosis would be most appropriate for a client admitted with heart failure? Ineffective tissue perfusion Acute pain Risk for denial Impaired gas exchange

ineffective tissue perfusion

Which of the following is not a true statement about intimate partner violence? One in every three women worldwide is a victim of intimate partner violence at some point in her life. Intimate partner violence is based on the abuser's need to maintain power and control over the victim. The annual cost in the United States for intimate partner violence exceeds $4 billion for medical and health services, and over $1 billion for lost productivity. Intimate partner violence is caused by the "victim" refusing her husband's desire for sex.

intimate partner violence is caused by the victim refusing her husband's desire for sex

A client arrives at the clinic accompanied by her husband. When the client is in the examination room she says to the nurse, "He loves me so much. He only lets me go out when he is with me because he says other men look at me." What type of behavior is this husband exhibiting? Intimidation Coercion Isolation Privilege

isolation

The emergency department nurse is assessing a female client with traumatic injuries. To assess whether or not the client's injuries have resulted from abuse, which question would be most appropriate for the nurse to ask the client? "Is your partner being mean to you?" "Why do you think your husband has beaten you?" "It looks like someone has hurt you. Tell me about it." "Can you describe the person who did this to you?

it looks like someone has hurt you tell me about it

In order to palpate an apical pulse when performing a cardiac assessment, where should the nurse place the fingers? left midclavicular line at the third intercostal space right of the midclavicular line at the third intercostal space left midclavicular line at the fifth intercostal space right of midclavicular line at the fifth intercostal space

left midclavicular line at the fifth intercostal space

A client with hypothyroidism is admitted to the medical unit. The nurse would expect to assess which signs/symptoms? Select all that apply. lethargy constipation cool skin lower systolic blood pressure weight loss

lethargy constipation cool skin lower systolic blood pressure

A client complains of feeling like he is slowly losing his central vision. The nurse knows this symptom could represent macular degeneration open-angle glaucoma hemianopsia retinal detachment

macular degeneration

The nurse is interviewing Mr. Jenkins and, due particularly to his nervous affect and his reaction when his son is mentioned, suspects potential elder abuse. In assessing Mr. Jenkins, the nurse should focus exclusively on the physical examination, as elder abuse is primarily physical in nature. make sure that the assessment includes questions to ensure that Mr. Jenkins has access to food and needed medication. ask to speak to Mr. Jenkins' son directly, to ask him candidly about the potential abuse. keep in mind that elder abuse is usually reported, indicating that Mr. Jenkins is not likely a victim.

make sure that the assessment includes questions to ensure that Mr. Jenkins has access to food and needed medication.

A nurse is caring for a client admitted with neck pain. The client is febrile. What is the most likely medical diagnosis for this client? Migraine Meningitis Cervical fracture Measles

meningitis

A female client visits the clinic and tells the nurse that she frequently experiences severe recurring headaches that sometimes last for several days and are accompanied by nausea and vomiting. The nurse determines that the type of headache the client is describing is a migraine headache. cluster headache. tension headache. tumor-related headache.

migraine headache.

The nurse identifies which of the following as risk factors for spiritual distress? (Select all that apply.) negative life changes natural disasters chronic illnesses experiencing a new love life ability to forgive

negative life changes natural disasters chronic illnesses

The nurse and a client are developing a transcultural nursing care plan related to hypertension. The nurse will include in the plan educational materials given to all clients with hypertension. nutritional information specifically for the client's culture. contact number for the cardiac rehabilitation program at the medical center. referral to a cardiac specialist.

nutritional information specifically for the client's culture.

The nurse assesses a client who has ventricular enlargement. The nurse palpates the left parasternal area but cannot feel the ventricle. Which underlying condition does this client likely have? obstructive pulmonary disease ischemic heart disease arrhythmia peripheral vascular disease

obstructive pulmonary disease

Which of the following describes a condition characterized by abnormal spongy bone formation around the stapes? Otosclerosis Middle ear infection Chronic otitis media Otitis externa

otosclerosis

While assessing an adult client's head and neck, the nurse observes asymmetry in front of the client's ear lobes. The nurse refers the client to the physician because the nurse suspects the client is most likely experiencing a/an enlarged thyroid. lymph node abscess. neurologic disorder. parotid gland enlargement.

parotid gland enlargement

The nurse knows that nonverbal communication can be difficult to interpret among of people of different cultures because of the many variations. These variations include which of the following? (Select all that apply.) patterns of space eye contact hand gestures beliefs touch

patterns of space eye contact hand gestures touch

After assessing pitting edema below the knee in a client, the nurse would suspect that which artery may be occluded? popliteal iliofemoral saphenous communicating

popliteal

The nurse is preparing to examine an adult client's eyes, using a Snellen chart. The nurse should position the client 609.6 cm (20 ft) away from the chart. ask the client to remove his glasses. ask the client to read each line with both eyes open. instruct the client to begin reading from the bottom of the chart.

position the client 609.6 cm (20 ft) away from the chart.

Which terms refers to the progressive hearing loss associated with aging? Presbycusis Exostoses Otalgia Sensorineural hearing loss

presbycusis

A client expresses to the nurse visiting her home that her husband has threatened to kill her. The nurse understands that threats of harm and intimidation are which type of abuse? Psychological Economic Physical Sexual

pychological

A nursing instructor is teaching the importance of religion and spirituality to clients. The instructor identifies which of the following as the first step in promoting the spiritual aspect of holistic care? reflect on one's own spiritual dimension consult a supervisor take a training class to increase knowledge read the Bible

reflect on one's own spiritual dimension

Shared practices and rituals used to express one's faith can be called religion. spirituality. denomination. philosophy.

religion

Photoreceptors of the eye are located in the eye's ciliary body. lens. retina. pupil.

retina

lient asks how long the hospitalization will be since family and friends will not be able to visit because of the distance to travel. Which nursing diagnosis should be identified for this client's concern? Anxiety Hopelessness RC: Depression Risk for social isolation

risk for social isolation

A nurse provides care for a client who experiences anxiety, gastrointestinal complaints, and a fear of being poisoned or killed. The nurse recognizes this as which culture-bound syndrome? Susto Rootwork Mal de ojo Empacho

rootwork

When the nurse asks a client of African origin for the reason for coming to the health clinic, the client responds by saying, "Bad blood." The nurse recognizes that the client is most likely referring to which of the following? Low blood volume due to diet High blood pressure Sexually transmitted infection Sudden collapse preceded by dizziness

sexually transmitted infection

To give spiritual care to a client, it is not necessary for the nurse to recognize that depression may result from unmet religious group expectations. share the same religious beliefs as the client to provide effective spiritual care. be aware of personal religious beliefs and biases in order to provide effective spiritual care. prepare to help clients seek a source of strength and hope from their own (clients') religion and spiritual beliefs.

share the same religious beliefs as the client to provide effective spiritual care.

A nursing student realizes that which of the following is an important variation of communication? silence beliefs spirituality death rituals

silence

A client is admitted to the health care facility after sustaining a crushing injury to the right eye. The nurse should anticipate abnormal results for which vision test? Accommodation Six cardinal positions of gaze Pupillary reaction to light Position and alignment of the eyeballs

six cardinal positions of gaze

The nurse is caring for a 63-year-old client who can neither read nor speak English. What would be the appropriate chart to use to assess this client's vision? Allen Snellen E Ishihara PERRLA

snellen E

The roof of the oral cavity of the mouth is formed by the anterior hard palate and the teeth. gums. muscles. soft palate.

soft palate

Which of the following is true about violence against women? Verbal attacks by a husband are not considered violence against a wife. Some experts purport that cultural attitudes influence violence. Between married couples, there is no behavior that meets the criteria for rape. Violence against women is a recent development in the United States.

some experts purport that cultural attitudes influence violence

The nurse is assessing a client's ability to employ coping mechanisms when given a diagnosis of cancer. What type of assessment would be beneficial for the nurse to use? Spiritual assessment Cultural assessment Physical assessment Mini-Mental status assessment

spiritual assessment

Matters of the human soul are referred to as what? Culture Ethnicity Values and beliefs Spirituality

spirituality

The client states "What does it matter? I have no purpose in this life." This client is questioning what? Religion Spirituality Cultural beliefs Cultural practices

spirituality

The path one pursues in the search for life's meaning and purpose. Religion Spirituality Denomination Philosophy

spirituality

Which characteristic demonstrates the nurse's understanding and practice of cultural competent nursing? Select all that apply. states, "I'm really interested in learning about the dietary practices of the Hasidic Jews." recognizes the need to provide all clients holistic care states, "I need to know all there is to know about the culture of my Native American clients." provides care using an established set of cultural skills understands that a person's culturally values undergo regular change

states, "I'm really interested in learning about the dietary practices of the Hasidic Jews." recognizes the need to provide all clients holistic care

A 12-year-old presents to the clinic with his father for evaluation of a painful lump in the left eye. It started this morning. The client denies any trauma or injury. There is no visual disturbance. Upon physical examination, there is a red raised area at the margin of the eyelid that is tender to palpation; no tearing occurs with palpation of the lesion. Based on this description, what is the most likely diagnosis? Dacryocystitis Chalazion Stye Xanthelasma

stye

The nurse reviews data collected while completing a spiritual assessment with a client. Which data should the nurse focus as the primary source of information for this assessment? Objective Subjective Medical record Secondary sources

subjective

Healthy People identifies a goal of reducing the annual rate of rape or attempted rape, and other sexual assaults. What action best facilitates this goal? Increased electronic surveillance Reducing wage gaps between men and women Teaching children about healthy touch Teaching women about the danger of men

teaching children about healthy touch

A client visits the clinic and tells the nurse that he is depressed because of a recent job loss. He complains of dull, aching, tight, and diffuse headaches that have lasted for several days. The nurse should recognize that these are symptoms of cluster headaches. tumor-related headaches. migraine headaches. tension headaches.

tension headaches

When performing a cultural assessment, an important point to remember would be that definitions of family differ to use first names of those you are speaking to that alternative therapies are reasons for seeking care that the cultural/ethnic background is evident in the client's appearance

that definitions of family differ

The nurse is educating a client from another country about the medications they will be taking. The client continually interrupts the nurse during the conversation. When the nurse considers the many ways cultural differences can affect communication, how should the nurse interpret the client's interruptions? This client is impolite and does not know good manners. The client believes that what they have to say is more important than what the nurse is saying. The client is deeply engaged in the conversation. The client believes that they know more about the medications than the nurse does.

the client is deeply engaged in the conversation

The nurse is caring for a family in which the elderly mother has been a victim of abuse and neglect by her 48-year-old son. Which of the following would be most important for the nurse to keep in mind before interviewing the family? A top nursing priority will be to legally remove the son from the home. The main focus of the nurse's actions should be on improving the elderly mother's self-esteem The nurse must allow the elderly mother to decide if she wants to leave the situation or not. Placement for the elderly woman in a nursing home within the community.

the nurse must allow the elderly mother to decide if she wants to leave the situation or not

The nurse is auscultating the heart sounds of an adult client. To auscultate Erb point, the nurse should place the stethoscope at the second intercostal space at the right sternal border. third to fifth intercostal space at the left sternal border. apex of the heart near the midclavicular line (MCL). fourth or fifth intercostal space at the left lower sternal border.

third to fifth intercostal space at the left sternal border.

An adult client visits the clinic complaining of a sore throat. After assessing the throat, the nurse documents the client's tonsils as 4+. The nurse should explain to the client that 4+ tonsils are present when the nurse observes tonsils that are touching the uvula. visible upon inspection. touching each other. midway between the tonsillar pillars and uvula.

touching each other.

When preparing to examine a client's sclera and conjunctiva during an eye examination, the nurse should instruct the client to move both eyes to look in which direction? Up Down To the right To the left

up

A teenager is brought to the clinic for a sports physical examination. The client states plans to play goalie on the community soccer team. What is the most important teaching opportunity presented for this client? Use of safety equipment Prevention of knee injuries Prevention of head injuries Use of correct foot gear

use of safety equipment

A client has a brownish discoloration of the skin of both lower legs. What should the nurse suspect is occurring with this client? atherosclerosis arterial insufficiency venous insufficiency deep vein thrombosis

venous insufficiency

During a physical examination, the nurse detects warm skin and brown pigmentation around an adult client's ankles. The nurse suspects that the client may be experiencing venous insufficiency. arterial occlusive disease. venous ulcers. ankle edema.

venous insufficiency.

When planning care for a client with an inner ear infection, the nurse will need to include interventions for which of the following potential problems? Rhinorrhea Fever Headache Vertigo

vertigo

The rich blood supply of the nose serves to help propel moist air to the body. propel debris to the throat. filter large particles from the air. warm the inspired air.

warm the inspired air.

A 4-year-old has been sexually abused. As you watch her play with anatomically correct dolls, she inserts the male doll's penis into the female doll's mouth. What is your best response to this action? "Be careful; you'll hurt the doll that way." "Are you playing a game from television." "What are the dolls doing?" "Nice dolls don't do that. Why are you playing that way?"

what are the dolls doing

What aspects of culture are relevant when conducting a health assessment on a foreign-born client admitted for surgery? Select all that apply. whether there are any existing language barriers nutritional or dietary considerations alternative medicine practices reasons for immigrating to the United States age when immigration to the United States occurred

whether there are any existing language barriers nutritional or dietary considerations alternative medicine practices


Conjuntos de estudio relacionados

Exam 4 - Evidence for Evolution & Origin of Species

View Set

chapter 6 correlational methods and statistics

View Set

Brokerage Relationships: Laws and Practice (National)

View Set

Chapter 09 Assignment: Attracting and Retaining the Best Employees

View Set

Chapter 4 - Fulfillment Process Study Questions

View Set